ดูหนึ่งข้อความ
  #1  
Old 19 มกราคม 2014, 13:02
math magic's Avatar
math magic math magic ไม่อยู่ในระบบ
จอมยุทธ์หน้าใหม่
 
วันที่สมัครสมาชิก: 05 กุมภาพันธ์ 2012
ข้อความ: 74
math magic is on a distinguished road
Default เรื่อง mod ครับ

ผมสงสัยว่า ถ้า $13^400$$\equiv$ 1(mod 1000)

สมมุติว่า $13^398$$\equiv$ x(mod n)
$13^400$$\equiv$ 169x(mod n)
แต่$13^400$$\equiv$ 1(mod n)
ดังนั้น 169x$\equiv$ 1(mod n)

x = ? ทำอย่างนี้ได้ไหมครับ
__________________
หากวันไหนรู้สึกท้อแท้กับช่วงเวลาที่ต้องฝึกฝน
บอกกับตัวเองให้อดทนลำบากตอนนี้ เพื่ออนาคตที่ดีในวันหน้า

I hated every minute of
training, but I said, "Don't
quit. Suffer now and live
the rest of your life as a champion."


19 มกราคม 2014 13:03 : ข้อความนี้ถูกแก้ไขแล้ว 2 ครั้ง, ครั้งล่าสุดโดยคุณ math magic
ตอบพร้อมอ้างอิงข้อความนี้